Find the mean and the mean absolute deviation of each data set

Answers

Answer 1

To find the mean and mean absolute deviation of a data set, you need to follow these steps:

1. Find the mean: To find the mean of a data set, add up all of the values in the set and then divide that sum by the number of values in the set. For example, if your data set is {2, 4, 6, 8, 10}, you would add up all of the values (2+4+6+8+10=30) and then divide that sum by the number of values (5). So the mean of this data set is 30/5 = 6.

2. Find the mean absolute deviation:

To find the mean absolute deviation of a data set, you first need to find the absolute deviation of each value in the set from the mean.

To do this, subtract the mean from each value in the set (for example, if your data set is {2, 4, 6, 8, 10} and the mean is 6, you would subtract 6 from each value: 2-6=-4, 4-6=-2, 6-6=0, 8-6=2, 10-6=4).

Then, take the absolute value of each of these differences (|-4|=4, |-2|=2, |0|=0, |2|=2, |4|=4). Finally, find the mean of these absolute deviations by adding them up and dividing by the number of values in the set.

For the example data set above, the mean absolute deviation is (4+2+0+2+4)/5 = 2.4.

Learn more about mean absolute deviation: https://brainly.com/question/3998118

#SPJ11


Related Questions

Find the sum of the convergent
∑ 24/n(n+2)
n = 1

Answers

the sum of the convergent series is 8.

To find the sum of the convergent series ∑(24/n(n+2)) where n starts at 1, we can re-write the given expression as a partial fraction decomposition:

24/n(n+2) = A/n + B/(n+2)

Solving for A and B, we find that A = 12 and B = -12. So the expression becomes:

12/n - 12/(n+2)

Now, we can compute the sum for the given series:

∑[12/n - 12/(n+2)] from n = 1 to infinity

As this is a telescoping series, most of the terms will cancel out. We are left with:

12/1 - 12/3 + 12/2 - 12/4 + ... + 12/∞ - 12/(∞+2)

The sum converges to:

12 - 12/3 = 12 * (1 - 1/3) = 12 * 2/3 = 8

To learn more about convergent series click here

brainly.com/question/15415793

#SPJ11

3cm on a map represents a distance of 60 if the scale is expressed in the ratio 1:n then n

Answers

3:60 = 1:n, so n = 20

Find the value of x.

Answers

Step-by-step explanation:

180° + 42° + x = 360°

222° + x = 360°

x = 138°

Answer:

138

Step-by-step explanation:

Since a circle is 360 degrees and it is split in half, each half would equal 180 degrees so you would subtract 42 from 180

The Mars Rover Curiosity is sending signals that it is driving into a crater at an angle of depression of 53°.



If the rover covers a horizontal distance of 110 meters, what vertical distance has it traveled? Round your answer to the nearest thousandth

Answers

The vertical distance traveled by the rover is approximately 140.784 meters.

What is the vertical distance traveled by Mars Rover Curiosity?

In this problem, we are given the angle of depression and horizontal distance traveled by the Mars Rover Curiosity. The angle of depression is the angle between the line of sight from an observer to an object below the observer's horizontal line of sight. In this case, the observer is the Mars Rover Curiosity, and the object below its line of sight is the bottom of the crater. The horizontal distance traveled by the rover is 110 meters.

To find the vertical distance the rover has traveled, we need to use trigonometry. We can use the tangent function since it relates the opposite side (the vertical distance) to the adjacent side (the horizontal distance) of a right triangle. Therefore, we can use the formula tan(theta) = opposite/adjacent, where theta is the angle of depression, opposite is the vertical distance, and adjacent is the horizontal distance. Rearranging this formula, we get opposite = adjacent * tan(theta).

Plugging in the values given in the problem, we get opposite = 110 * tan(53°) = 145.911 meters (rounded to the nearest thousandth). Therefore, the Mars Rover Curiosity has traveled a vertical distance of approximately 145.911 meters into the crater.

This would be:

Let h be the vertical distance traveled by the rover. Then we have:

tan(53°) = h/110

Solving for h, we get:

h = 110 * tan(53°) ≈ 140.784 meters

Therefore, the vertical distance traveled by the rover is approximately 140.784 meters.

Learn more about angle of depression

brainly.com/question/13514202

#SPJ11

Can someone help me asap? It’s due today!! I will give brainliest if it’s correct.

Answers

Answer:

im pretty sure its A = 10

Adriel decides to research the relationship between the length in inches and the


weight of a certain species of catfish. He measures the length and weight of a number


of specimens he catches then throws back into the water. After plotting all his data,


he draws a line of best fit. What does the slope of the line represent?

Answers

The slope of the line  represents the rate of change in weight for every unit increase in length of the catfish.

How to find the slope of line?

The slope of the line of best fit in this scenario represents the rate of change or the relationship between the length and weight of the catfish. Specifically, the slope indicates the change in weight of the catfish for every unit increase in length.

If the slope is positive, it means that as the length of the catfish increases, its weight also tends to increase. If the slope is negative, it means that as the length of the catfish increases, its weight tends to decrease.

For example, if the slope of the line of best fit is 2, it means that for every one-inch increase in length, the weight of the catfish tends to increase by two pounds. Similarly, if the slope is -1, it means that for every one-inch increase in length, the weight of the catfish tends to decrease by one pound.

In summary, the slope of the line of best fit represents the relationship between the two variables being studied, in this case, the length and weight of the catfish.

Learn more about slope

brainly.com/question/31219359

#SPJ11

(2 points) Find the Laplace transform of f(t) = -1, 0 3 { F(x) = (2 points) Find the Laplace transform of f(t) = S (t - 5), 0 5 - F(3) = )

Answers

Laplace transform of f(t) = -1, 0 3 { F(x)

The Laplace transform of f(t) = S(t - 5), 0, 5 - F(3) is F(s) = (1/s) [tex]e^{(-5s)[/tex] - (1/3) [tex]e^{(-15)[/tex].

Laplace transform:

The Laplace transform of a function f(t) is given by:
F(s) = ∫[0,∞) e^(-st) f(t) dt
where s is a complex variable.
Using this formula, we can find the Laplace transform of f(t) as follows:
F(s) = ∫[0,∞) e^(-st) f(t) dt
    = ∫[0,∞) e^(-st) (-1) dt + ∫[0,∞) e^(-st) (0) dt + ∫[0,∞) e^(-st) (3) dt
    = -1/s + 0 + 3/s
    = (2/s) - (1/s)
Therefore, the Laplace transform of f(t) = -1, 0, 3 is F(s) = (2/s) - (1/s).
Now, let's move on to the second part of the question.

We need to find the Laplace transform of f(t) = S(t - 5), 0, 5 - F(3).
Here, S(t - 5) is the Heaviside step function, which is defined as:
S(t - 5) = 0, for t < 5
        = 1, for t ≥ 5
Using the Laplace transform formula, we can write:
F(s) = ∫[0,∞) e^(-st) S(t - 5) dt
Since S(t - 5) is equal to 0 for t < 5, we can split the integral into two parts:
F(s) = ∫[0,5) [tex]e^(-st)[/tex]S(t - 5) dt + ∫[5,∞) [tex]e^(-st)[/tex] S(t - 5) dt
The first integral is equal to 0, since S(t - 5) is 0 for t < 5.
For the second integral, we can use the fact that S(t - 5) = 1 for t ≥ 5. So, we get:
F(s) = ∫[5,∞) e^(-st) dt
    = [-1/s e^(-st)]_[5,∞)
    = (1/s) [tex]e^(-5s)[/tex]
Finally, we need to find F(3). Substituting s = 3 in the Laplace transform, we get:
[tex]F(3) = (1/3) e^(-15)[/tex]
Therefore, the Laplace transform of f(t) = S(t - 5), 0, 5 - F(3) is F(s) = (1/s) [tex]e^(-5s) - (1/3) e^(-15).[/tex]

To know more about Laplace transform:

https://brainly.com/question/31041670

#SPJ11

Mortgage payments Principal : $ 180,000.00 Interest Rate Monthly Payment How much money will be spent in interest alone over the course of the 3.5 % 30 - year mortgage described in the table ? 3.5% 5% $808 $966 $ 1079 6% A. $110,880 B. $6,300 C. $180,000 D. $ 290,880

Answers

Answer:

To calculate the amount of money spent in interest alone over the course of a 30-year mortgage, we can use the formula:

Total Interest = (Monthly Payment x Number of Payments) - Principal

For a 3.5% 30-year mortgage with a principal of $180,000, the monthly payment can be calculated using the formula:

Monthly Payment = (Principal x Monthly Interest Rate) / (1 - (1 + Monthly Interest Rate)^(-Number of Payments))

where Monthly Interest Rate = Annual Interest Rate / 12, and Number of Payments = 30 years x 12 months per year = 360.

Plugging in the values, we get:

Monthly Payment = (180,000 x 0.0035) / (1 - (1 + 0.0035)^(-360)) = $808.28

Using this monthly payment, we can calculate the total interest over the 30-year period:

Total Interest = ($808.28 x 360) - $180,000 = $101,020.80

Therefore, the correct answer is A. $110,880 (which is not one of the options given).

Select all the polygons that can be formed by the intersection of a plane and a cylinder, either parallel or perpendicular to the base?

Answers

The intersection of a plane and a cylinder can result in the following polygons when the plane is either parallel or perpendicular to the base:

Oa Rectangle

Oc Square

Od Triangle

What are the polygons that can be formed

The angle and position of the plane relative to the cylinder intersect determines a myriad of shapes not limited to just one. Herein are the descriptions for a few that may arise:

Rectangle: If a plane intersects the cylinder but remains parallel to its base, the resulting outline shall be rectangular. This is due to the aforementioned plane cutting through the lateral surface of the circumference, forming two equal lines--thereby shaping a closed loop in an angular fashion.

Square: The intersection of a plane perpendicularly bisecting the base of a cylindrical object will conjure up a square shape congruent to the cylinders. In other words, a perfectly squared compartment matching with the pre-existing edges of the bottom curve of the cylinder.

Read more on polygons here: https://brainly.com/question/1592456

#SPJ1

6 Which graph best represents a quadratic function with a range of all
real numbers greater than or equal to 3?
F
G
H
H
P
J

Answers

The fourth graph best represents a quadratic function with a range of all real numbers greater than or equal to 3

The graph that best represents a quadratic function with a range of all real numbers greater than or equal to 3 is a graph that opens upward and has a vertex at the point (h, k), where k is the minimum value of the function.

Since the range is all real numbers greater than or equal to 3, the minimum value occurs at or above 3.

Therefore, the vertex of the quadratic function lies on or above the horizontal line y = 3.

Hence, the fourth graph best represents a quadratic function with a range of all real numbers greater than or equal to 3

To learn more on Graph click:

https://brainly.com/question/17267403

#SPJ1

Please hurry I need it ASAP

Answers

Answer:

20

Step-by-step explanation:

(x-3)+(8x+3)=180

9x=180

x=20

What is the volume of a sphere with a radius of 2.5? answer in terms of pi

options:
-20 5/6π
-25π
-8 1/3π
-15 5/8π

Answers

[tex]8 \frac{1}{3} \pi[/tex]

Step-by-step explanation:

volume of a sphere = 4/3 pi r²

r = 2.5

4/3× pi× 2.5² = 25/3pi

25/3 as a mixed number is 8 and 1/3

therefore rhe answer is 8 and 1/3 pi

N equation for the depreciation of a car is given by y = A(1 – r)t , where y = current value of the car, A = original cost, r = rate of depreciation, and t = time, in years. The value of a car is half what it originally cost. The rate of depreciation is 10%. Approximately how old is the car?

3. 3 years

5. 0 years

5. 6 years

6. 6 years

Answers

the car is approximately 6.6 years old. The closest option provided is 6 years, so the answer is (C) 6 years.

A car's original value depreciates by 10% per year. If the current value of the car is half of its original value, approximately how old is the car?

Given:

y = A(1 – r)t

The value of a car is half what it originally cost, which means:

y = 1/2 A

The rate of depreciation is 10%, which means:

r = 0.1

Substituting these values in the equation, we get:

1/2 A = A(1 – 0.1)t

Simplifying, we get:

1/2 = 0.9t

Solving for t, we get:

t = ln(1/2) / ln(0.9) ≈ 6.6 years

Learn more about value

brainly.com/question/10416781

#SPJ11

Please help :D

A. Explain how to make a prediction based on the probability of an event.

B. Then, give an example in which predictions are made based on probabilities

Answers

This prompt is about probability. The answers are given as follows;

How can one  make prediction based on the probability of an event  ?

Identifying the   probability of an event is crucial to making predictions based on its likelihood. T his involves calculating the probability either through historical data or experimentation.

Once determined, utilizing this value enables one to make future predictions regarding the occurrence of such events; for instance, 80% probability of precipitation tomorrow implies an 80% chance of rain.

Calculating probabilities has proven essential to sports betting because it helps bookmakers given some degree of foresight on which teams are going to win specific games or tournaments. Operating under the premise that there will always be two probable outcomes (either one side wins while another loses), these bookmakers could assign numerical values on what percentage they deem worthy enough for each team's chances.

Subsequently, using precise mathematical formulas and equations, bettors assess wagering-related uncertainties based on these predetermined likelihoods before deciding whether or not they should place money bets.

Learn more about probability:
https://brainly.com/question/29381779
#SPJ1

which statement is true about the mean of the data set?

Answers

Step-by-step explanation:

Mean is less than 8

(1 + 1 + 6*8 + 10 ) / 9 = mean = 6.7

Answer:

A: The mean in less than 8

Step-by-step explanation:

Mean: Average

How to find the mean?

1. Add ALL the numbers given

     1: has 2 dots     8: has 6 dots     10: has 1 dot

so  2+6+1= 9

2. divide the result of point 1. by the amount of numbers given.

     9/3= 3

3.

  Numbers:

    1-2-3-4-5-6-7-8-9   3 is before 8 so it means it's less than 8.

Cher is making hotdogs for her coworkers to celebrate their 5 year
anniversary. Hotdogs come in packs of 6, while the buns come in
packs of 10. How many hotdogs should Cher cook to have the
smallest number of hotdogs and hotdog buns?

Answers

To have the smallest number of hotdogs and hotdog buns, we have to find the least common multiple of 6 and 10. The LCM is the smallest number that is a multiple of both 6 and 10.

To find the LCM, we can list the multiples of each number until we find the first multiple that they have in common:

Multiples of 6: 6, 12, 18, 24, 30, 36, 42, 48, 54, 60 and ongoing.
Multiples of 10: 10, 20, 30, 40, 50, 60, 70, 80, 90, 100 and ongoing.

The first multiple that they have in common is 30, so the LCM of 6 and 10 is 30.

Since hotdogs come in packs of 6 and buns come in packs of 10, we need to make sure that the number of hotdogs we cook is a multiple of 6 and the number of buns we buy is a multiple of 10. Therefore, we need to cook 30 hotdogs so that we can buy 3 packs of hotdogs (18 hotdogs) and 3 packs of buns (30 buns).

Therefore, Cher should cook 30 hotdogs to have the smallest number of hotdogs and hotdog buns.

PLEASE HELP I NEED HELP QUICK!!!

Answers

There are 720 different arrangements of the six children possible when Ben can't sit next to Dan.

There are 720 different arrangements of the six children possible.

The key to solving this problem is to recognize the fact that there are six children and six chairs, so each child has one and only one chair. This means that for each position in the row, one child must be placed in the chair.

To solve this problem we can use the permutation formula for "n objects taken r at a time without repetition," which is: n!/(n-r)!

In this case, n is 6 (the number of children) and r is 6 (the number of chairs). So, 6!/(6-6)! = 6!/(0!) = 6!/1 = 6! = 720.

Therefore, there are 720 different arrangements of the six children possible when Ben can't sit next to Dan.

To learn more about the permutation and combination visit:

https://brainly.com/question/28065038.

#SPJ1

A large research organization wants to recruit graduate secretaries/typists from two commercial institutes. The personnel manager of the organization gave a typing test to 35 graduating students from each of the commercial institutes and observed that the mean of the first group was 65 words per minute with a S1 = 15. The mean of the second group was 70 words per minute with S2 = 10. Using a 1% level of significance, can we say there is a significant difference between the mean scores of the graduates in the two commercial institutes?

Answers

In summary, we can say that there is a significant difference in the mean scores of the graduates in the two commercial institutes.

To determine if there is a significant difference between the mean scores of the graduates in the two commercial institutes, we can perform an independent samples t-test. Here's how to approach it:

Step 1: State the hypotheses:

Null hypothesis (H0): The mean scores of the graduates in the two commercial institutes are equal.

Alternative hypothesis (Ha): The mean scores of the graduates in the two commercial institutes are significantly different.

Step 2: Set the significance level:

The significance level (α) is given as 1%, which corresponds to a critical value of 0.01.

Step 3: Calculate the test statistic:

The test statistic for an independent samples t-test is calculated using the following formula:

t = (mean1 - mean2) / √[(S1^2 / n1) + (S2^2 / n2)]

Given:

Mean of the first group (mean1) = 65

Standard deviation of the first group (S1) = 15

Sample size of the first group (n1) = 35

Mean of the second group (mean2) = 70

Standard deviation of the second group (S2) = 10

Sample size of the second group (n2) = 35

Plugging in the values, we can calculate the test statistic:

t = (65 - 70) / √[(15^2 / 35) + (10^2 / 35)]

t = -5 / √[225/35 + 100/35]

t = -5 / √[325/35]

t ≈ -5 / 1.787

t ≈ -2.8 (rounded to one decimal place)

Step 4: Determine the critical value and compare:

Since the significance level (α) is 1%, the critical value for a two-tailed test is ±2.61 (obtained from a t-distribution table or a statistical software).

Since the calculated test statistic (-2.8) is greater than the critical value (-2.61) in absolute value, we reject the null hypothesis.

Step 5: Interpret the result:

Based on the test, we have sufficient evidence to conclude that there is a significant difference between the mean scores of the graduates in the two commercial institutes at the 1% level of significance.

In summary, we can say that there is a significant difference in the mean scores of the graduates in the two commercial institutes.

Learn more about null hypothesis,

https://brainly.com/question/25263462

#SPJ11

Let f(x)= x⁴ - 6x³ - 60x² + 5x + 3. Find all solutions to the equation f'(x) = 0. As your answer please enter the sum of values of x for which f'(x) = 0.

Answers

The answer is 2, which represents the sum of the values of x for which f'(x) = 0.

How to find critical points?

To find the critical points of f(x), we need to find the derivative of f(x):

f(x) = x⁴ - 6x³ - 60x² + 5x + 3f'(x) = 4x³ - 18x² - 120x + 5

Setting f'(x) = 0 and solving for x, we get:

4x³ - 18x² - 120x + 5 = 0

We can use the Rational Root Theorem to find possible rational roots of the equation. The possible rational roots are:

±1, ±5/4, ±3/2, ±5, ±15/4, ±3, ±15, ±1/4

We can use synthetic division or long division to check which of these roots are actually roots of the equation. We find that the only real root is x = 5/4, and it has multiplicity 2.

The sum of the values of x for which f'(x) = 0 is simply the sum of the critical points of f(x). In this case, we only have one critical point: x = 5/4.

5/4 + 5/4 = 10/4 = 2.

We first find the derivative of the given function and set it equal to zero to find the critical points. We use the Rational Root Theorem to find the possible rational roots of the equation, and then we use synthetic division or long division to check which of these roots are actually roots of the equation. In this case, we find that the only critical point of the function is x = 5/4 with multiplicity 2.

Learn more about Critical points

brainly.com/question/31017064

#SPJ11

Let u= (3, -7) and v = (-3.1). Find the component form and magnitude (length) of the vector 2u - 4v.

Answers

I think there might be a typo in the question - it looks like there's a missing second coordinate for vector v. Assuming that the second coordinate for v is also -7, here's the solution:

First, let's find the component form of 2u - 4v:

2u = 2(3,-7) = (6,-14)
4v = 4(-3,-7) = (-12,-28)

So 2u - 4v = (6,-14) - (-12,-28) = (6+12, -14+28) = (18,14)

Therefore, the component form of 2u - 4v is (18,14).

To find the magnitude of (18,14), we can use the Pythagorean theorem:

|(18,14)| = sqrt(18^2 + 14^2) = sqrt(360) ≈ 18.97

So the magnitude (length) of the vector 2u - 4v is approximately 18.97.
To find the component form of the vector 2u - 4v, we'll first perform scalar multiplication and then vector subtraction.

Scalar multiplication:
2u = 2(3, -7) = (6, -14)
4v = 4(-3, 1) = (-12, 4)

Vector subtraction:
2u - 4v = (6, -14) - (-12, 4) = (6 + 12, -14 - 4) = (18, -18)

So, the component form of the vector 2u - 4v is (18, -18).

To find the magnitude (length) of the vector, we'll use the formula: ||2u - 4v|| = √(x² + y²), where x and y are the components of the vector.

Magnitude = √((18)² + (-18)²) = √(324 + 324) = √(648) ≈ 25.46

The magnitude (length) of the vector 2u - 4v is approximately 25.46.

Learn more about Vector subtraction here: brainly.com/question/11865448

#SPJ11

The regular price of a red T-shirt is $6.93. Ernest has a coupon for $6.75 off. How much will Ernest pay for the T-shirt?

Answers

Answer:

18 cent

Step-by-step explanation:

This is a really easy problem. To solve it just subtract $6.93 and 6.75. You should get .18 cents if you did it correctly

Emilio saves 25% of the money he earns babysitting. he earns an average of $30 each week. which expression represents the change in emilio’s savings each week?

Answers

The expression that represents the change in Emilio's savings each week is $7.50.

How to find the Emilio savings?

Emilio saving 25% of the money he earns babysitting, which means that he saves a quarter of his earnings. This can be expressed mathematically as:

savings = 0.25 x earnings

where "savings" is the amount Emilio saves and "earnings" is the amount he earns each week.

Substituting the given value of Emilio's average weekly earnings of $30, we get:

savings = 0.25 x $30

savings = $7.50

Therefore, Emilio saves $7.50 each week.

Since the question asks for the change in Emilio's savings each week, the expression that represents this is simply:

$7.50

This means that Emilio's savings increase by $7.50 each week.

Learn more about  Saving

brainly.com/question/30004719

#SPJ11

You roll a six-sided number cube and flip a coin. What is the probability of rolling a number greater than 1 and flipping heads?

Answers

Answer:80%

Step-by-step explanation:

A certain painting was purchased for $15,000. its value is predicted to decay exponentially decreasing by 15% each year. which equation can be

used to predict t, the number of years it would take for the painting to have a value of $10,000?

a 10,000(0. 15)' = 15,000

b. 15,000(0. 15)' = 10,000

o g. 15,000(0. 85)' = 10,000

d. 10,000(0. 85)' = 15,000

Answers

The correct equation to predict the number of years it would take for the painting to have a value of $10,000 is 15,000(0.85)[tex]^{(t)}[/tex] = 10,000. The correct answer is option (c).

The initial value of the painting is $15,000, and its value is predicted to decay by 15% each year. This means that its value after t years can be represented by the equation:

V(t) = 15,000(0.85)[tex]^{(t)}[/tex]

We want to find the number of years it would take for the value to reach $10,000, so we set V(t) equal to 10,000 and solve for t:

10,000 = 15,000(0.85)[tex]^{(t)}[/tex]

Dividing both sides by 15,000 gives:

0.6667 = 0.85[tex]^{(t)}[/tex]

Taking the natural logarithm of both sides gives:

ln(0.6667) = t ln(0.85)

Solving for t gives:

t = ln(0.6667) / ln(0.85) = 2.294

So it would take approximately 2.294 years for the painting to have a value of $10,000. The right option is (c).

To know more about  equation click here

brainly.com/question/556412

#SPJ11

Im actually going to explode I hate geometry with a passion

Answers

Answer:  B   28√15

Step-by-step explanation:

They want you to find the Area of the quadrilateral.  But if you can find the area of 1 trangle then you can double it because the 2 triangles are congruent.

We know the 2 triangles are congruent from the theorem  (see diagram

We also know that  QR=TR=SR  They are all radius

Let's solve for triangle PRS

PR = hypotenuse = 10+7 =17

SR=7  radius

Use pythagorean to find PS

c²=a²+b²

17²=7²+b²

289 = 49 + b²

b²=289 -49

b² = 240

b=√240

b=[tex]\sqrt{16*15}[/tex]

b= 4√15  This is PS

Area of triangle = 1/2 bh       b=PS   h=7

Area of triangle = 1/2 (4√15)(7)

Area of triangle = (2√15)(7)

Area of triangle = 14√15

Area of quadrilateral = 2 (14√15)        > 2 triangles make the quadrilateral

Area of quadrilateral =  28√15

PLEASE HELP!! LIKE ASAPP

Answers

Answer:

12(8) + (1/2)(13)(20) + (1/4)π(8^2)

= 96 + 130 + 4π = 226 + 16π ft^2

= about 276.27 ft^2

Find the lateral area of the rectangular prism with height h, if the base of the prism is:



Square with the side 2 cm and h=125mm

Answers

The lateral area of the rectangular prism with base square with the side 2 cm and height 125 mm is 10,000 mm².

How to find the lateral area of rectangular prism?

To calculate the lateral area of a rectangular prism, we need to add up the areas of all its lateral faces.

In this case, the base of the prism is a square with side length 2 cm. Since there are four lateral faces on a rectangular prism, and each lateral face of the rectangular prism is a rectangle, we know that the length and width of each lateral face is equal to the height of the prism, which is 125 mm.

First, let's convert the side length of the base to millimeters to match the unit of the height:

2 cm = 20 mm

Now, we can calculate the lateral area of the rectangular prism as follows:

Lateral area = 4 x (length x height)

= 4 x (20 mm x 125 mm)

= 10,000 mm²

Therefore, the lateral area of the rectangular prism with base square with the side 2 cm and height 125 mm is 10,000 mm².

Learn more about rectangular prism

brainly.com/question/13162015

#SPJ11

Round all answers to the nearest cent. The revenue from the sale of x high-end cameras is given by: R(x)=1000x−5x 2a. What is the average change in revenue if production is changed from x=14 to x=17 ? $ b. What is the instantaneous rate of change in revenue at x=14?

Answers

The instantaneous rate of change in revenue at x=14 is $860 per camera.

The average change in revenue is $85 per camera [(ΔR/Δx) = 255/3].

a. The average change in revenue if production is changed from x=14 to x=17 is equal to the average rate of change of the revenue function over this interval:

Δx = 17 - 14 = 3

ΔR = R(17) - R(14) = (100017 - 517^2) - (100014 - 514^2) = $255

Therefore, the average change in revenue is $85 per camera [(ΔR/Δx) = 255/3].

b. The instantaneous rate of change in revenue at x=14 is equal to the derivative of the revenue function evaluated at x=14:

R(x) = 1000x - 5x^2

R'(x) = 1000 - 10x

R'(14) = 1000 - 10(14) = $860

Therefore, the instantaneous rate of change in revenue at x=14 is $860 per camera.

To learn more about instantaneous rate  visit: https://brainly.com/question/28837697

#SPJ11

A student is buying a shirt that has a regular price of $18. After using a coupon, the price of the shirt is 0. 75x, where x is the regular price of the shirt. The clerk includes 6% sales tax on the price of the shirt. How much change should the student receive if the student pays for the shirt using a $20 bill?

Answers

The student should receive $5.69 in change.

How to find the price?

The price of the shirt after the coupon is applied is 0.75 times the regular price, so:

Price of the shirt = 0.75x

If x = $18, then the price of the shirt is:

Price of the shirt = 0.75($18) = $13.50

The clerk adds 6% sales tax to the price of the shirt:

Sales tax = 6% of $13.50 = 0.06($13.50) = $0.81

So the total cost of the shirt is:

Total cost = $13.50 + $0.81 = $14.31

If the student pays with a $20 bill, the change the student should receive is:

Change = $20 - $14.31 = $5.69

Therefore, the student should receive $5.69 in change.

Learn more about sales price

brainly.com/question/29442509

#SPJ11

5+8(3+x) simplified please

Answers

Answer: 8x +29

Step-by-step explanation:

5+8(3+x)

5+8(x+3)

__________

5 + 8(x+3)

5+ 8x +25

_________

5+8x+ 24

29+8x

____

8x+29

Other Questions
What is a theme of how the death of patroclus roused achilles, the making of the arms, and the vengeance of achilles from the iliad?. During taylor's first test of a car with a mass of 250 grams, she recorded 10 seconds, 10.3 seconds, and 10.4 seconds for her 3 trials. what would be the mean value she would use to compare with the other cars? From the story The creation myths by the Igorots How Filipino culture is portrayed? www Trapped! Elevator Invented - August 2, 1743 Have you ever noticed how television shows are always trapping characters together on elevators? In fact, when you see people getting onto an elevator on a television show, you can be pretty sure that the elevator is going to stop between floors, especially if the people are fighting or don't like each other. It's a very common story device - probably because the results can be very interesting. In honor of the invention of the elevator, on August 2, 1743, and in honor of all that trapping, look at the trapped elevator below. See if you can find at least 30 items that begin with the letters "tr." www. TANGEN NTC 119 coy, inc. initially issued 360,000 shares of $1 par stock for $1,800,000 in 2019. in 2020, the company repurchased 36,000 shares for $360,000. in 2021, 18,000 of the repurchased shares were resold for $288,000. in its balance sheet dated december 31, 2021, coy, inc.'s treasury stock account shows a balance of: Find the sum:125 (base 7) + 256 (base 7) HELP DUE IN 5 min Area=? consider the switch shown below. suppose that all packets have the same fixed length and it requires one time slot to transfer a packet from an input port to an output port for all switching fabric. assume the switch fabric is a crossbar (more details in chapter 4.2 in the book) so that at most one packet can be transferred to a given output port in a time slot, but different output ports can receive packets from different input ports in a single time slot. what is the minimal number of time slots needed to transfer the packets shown from input ports to their output ports. instead, if the switching fabric is a bus, what is the minimum number of time slots required to transfer all the packets. explain your answer. What are strategies to improve living conditions at informal settlements help me Please asap in inserted the picture NUMBER 6 BTW Chris is using the expression 4x + 2 to represent the number of students in his gym class. There are four times as many students as basketballs, and there are two more students in the locker room. What does x represent? (4 points) H2 + Br2 2HBr. How many liters of hydrogen gas are needed to react with 9.0 g of bromine? Find the area of a circle with a radius of 4 m two ways. First, find it using the formula for the area of a circle. Then, find it by breaking the circle into equal sectors and rearranging the sectors as a parallelogram. Show all calculations. Use , instead of an approximation, in your answers. Round to the nearest tenth Can someone help me find if the function is increasing or decreasing and why the c intercept is (look at the photo) use the ratio test to determine whether the series is convergent or divergent. [infinity] 13^n N = 1 (n + 1)62n + 1 identify an.Evaluate the following limit.lim n [infinity]an + 1anSince lim n [infinity]an + 1an yesswj8tywg5r wdwgdwf6gqgdygdevfvyfsG 100-3(4. 25)-13-4(2. 99) SOMEONE PLSS HELP MEE THIS IS DIE TMRW!! The radius of a cylinder water tank is 6 Ft and its height is 11 ft what is the volume of the tank. find the volume of the solid obtained by rotating the region bounded by 2 =8 32? and 2 = -2y about the line x= 9. round to the nearest thousandth. You need to have $32,500 in 20 in years. you can earn an annual interest rate of 5 percent for the first 6 years, 5.6 percent for the next 5 years, and 6.3 percent for the final 9 years. how much do you have to deposit today?